LSAT and Law School Admissions Forum

Get expert LSAT preparation and law school admissions advice from PowerScore Test Preparation.

 Administrator
PowerScore Staff
  • PowerScore Staff
  • Posts: 8919
  • Joined: Feb 02, 2011
|
#49536
Complete Question Explanation
(The complete setup for this game can be found here: lsat/viewtopic.php?t=18147)

The correct answer choice is (E)

The question stem forms a W ___ Y block. This can occur under either template:
J01_Game_#3_#17_diagram 1.png
As R can arrive first, answer choice (E) is correct.
User avatar
 Dave Killoran
PowerScore Staff
  • PowerScore Staff
  • Posts: 5849
  • Joined: Mar 25, 2011
|
#19913
The following reply is in response to this question:
This question is about Game #3 June 2001.

For question 17, why was C selected. The question says what could be true. So, I interpret that as it can be true but it can be false. R MUST be in the third spot for the game to work properly. So, shouldn't B be the answer. Because either T or V could be in the sixth spot?

Or am I reading something wrong?

Thanks!

Thanks for the question! Let's first talk about what Could Be True means. It basically covers to type of answers:

  • 1. An answer that happens some of the time, but doesn't have to happen (so sometimes it might not occur)

    2. An answer that always occurs. This is rarely the answer on the LSAT, but it is possible because Must falls under the purview of Could.
So, what's happening in #17? First, the question stem forms a W ___ Y block. This block can occur under either template (please bear with me on the diagrams:

  • Template #1:

    ..... R ..... W ..... S ..... Y ..... T/V ..... Q ..... V/T

    ..... 1 ..... 2 ..... 3 ..... 4 ..... 5 ..... 6 ..... 7


    Template #2:

    ..... ( Q , R > V/T ) ..... W ..... S ..... Y ..... T/V

    ..... 1 ..... 2 ..... 3 ..... 4 ..... 5 ..... 6 ..... 7

So, let's look at the answers, and consider how they work with the Could Be True aspect here:

  • Answer choice (A): This can never occur—only Q or Y could be sixth.
    Answer choice (B): This can never occur—only Q or Y could be sixth.
    Answer choice (C): This can never occur—only Q, S, V, or T could be third.
    Answer choice (D): This can never occur—only Q, R, V, T, or W could be second.
    Answer choice (E): This is the correct answer—R can arrive first under Template #1.

A tough question because of the existence of the two templates. Please let me know if that helps. Thanks!

Get the most out of your LSAT Prep Plus subscription.

Analyze and track your performance with our Testing and Analytics Package.